Proving ln(x) using infinite series

Click For Summary
The discussion revolves around proving that the limit expression ln(x) = lim (n→∞) (x^(1/2^n) - 1) * 2^n is valid. The original poster struggles with applying L'Hôpital's rule and seeks assistance in manipulating the equation to avoid the indeterminate form of infinity times zero. A suggestion is made to simplify the problem by letting a = 2n, transforming the limit into a more manageable form. The conversation also touches on the relationship between the derived limit and the exponential function, with questions about how to interpret the resulting expressions. Overall, the thread emphasizes the need for clarity in approaching limits and understanding their connection to logarithmic and exponential functions.
darewinder
Messages
14
Reaction score
0

Homework Statement



Well we are given a series of steps done with the number "x" and in the end the end value is ln(x). Basically we are asked to prove why it isn't a coincedience

Homework Equations



I put the steps into an equation, but i can't prove it.

ln(x) =^{lim }_{n->inf} (x^\frac{1} {2^n} -1)*2^n


The Attempt at a Solution



Well plugging in gives me inf times 0 so i thought of solving it using Hopital's rule but i can't get to a form where it is 0/0. I tried factoring, rationalizing but i couldn't get anywhere. I would appreciate if you guys can help me do this little bit.

and I would also appreciate if anyone can show me how to come up with a sequence for this. Second part of the question askes me to come up with my own limit of a sequence to get ln(x). I just flipped changed the sine of - inside the brackets to make it + and added the minus sign to the -2n at the end, but that's like the same thing. So if you guys can give me some ideas it would be great! :)

Thank you
 
Last edited:
Physics news on Phys.org
Since n only occurs in 2n, you can simplify by letting a= 2n. Then the problem becomes showing that
ln(x)= \lim_{a\rightarrow \infty}(x^(1/a)-1)(a)

Now I would be inclined to "reverse" the function: If y= (x1/a-1)(a), then x= (y/a+ 1)a. Do you recognize the limit of that as a common formula for ey? And if x= ey, then y= ln(x).
 
tanks a bunch ill get back on you on the reverse function let me work it out. I am eating right now :) But the limit makes sense (argg why didn't i think of that)
 
hmm i have a question about this, x= (y/a+ 1)a.

Wouldn't x=1 when we take the limit as a>>inf?

Thanks for your help

never mind i see how x wouldn't equal to 1 because y has some "a" terms in it so we don't know the ratio. But i don't see how x = e^y. Is the x formula like an equation for the e function?
 
Last edited:
Question: A clock's minute hand has length 4 and its hour hand has length 3. What is the distance between the tips at the moment when it is increasing most rapidly?(Putnam Exam Question) Answer: Making assumption that both the hands moves at constant angular velocities, the answer is ## \sqrt{7} .## But don't you think this assumption is somewhat doubtful and wrong?

Similar threads

  • · Replies 1 ·
Replies
1
Views
2K
  • · Replies 4 ·
Replies
4
Views
1K
  • · Replies 5 ·
Replies
5
Views
2K
  • · Replies 34 ·
2
Replies
34
Views
3K
  • · Replies 22 ·
Replies
22
Views
4K
  • · Replies 7 ·
Replies
7
Views
2K
  • · Replies 2 ·
Replies
2
Views
2K
  • · Replies 3 ·
Replies
3
Views
2K
  • · Replies 4 ·
Replies
4
Views
2K
  • · Replies 48 ·
2
Replies
48
Views
5K